Q17

 
DevorahK415
Thanks Received: 0
Vinny Gambini
Vinny Gambini
 
Posts: 10
Joined: December 01st, 2019
 
 
 

Q17

by DevorahK415 Sat May 16, 2020 5:25 pm

Can I get a walk through on 17?

I got to framing it as follows 3 ways but couldn't figure it didn't must be true anything fo me.
Thanks
 
Laura Damone
Thanks Received: 94
Atticus Finch
Atticus Finch
 
Posts: 468
Joined: February 17th, 2011
 
 
 

Re: Q17

by Laura Damone Tue May 19, 2020 2:34 am

Sure thing! First, some background on the setup because you need some inferences to start q 17. Nobody confesses after W, and S comes after W, so S doesn't confess. Also, since nobody confesses after W, the last slot can't be a confession. That gives us this:

___ ___ ___ -C ___ ___ -C
___ ___ T ___ ___ ___ ___

We also know T must have exactly two confessions after it. Since 4 and 7 aren't confessions, 5 and 6 must be, giving us this:

___ ___ ___ -C C C -C
___ ___ T ___ ___ ___ ___

We also can infer that T comes before W since there must be confessions after T but can't be any after W. Since W must come before S, who didn't confess, W and S have to be in 6 and 7:

___ ___ ___ -C C C -C
___ ___ T ___ ___ W S

Since we know Z comes before both X and V, Z has got to be 1 or 2. If Z is 2, X and V, then have to be 4 and 5. If Z is 1, X and V could be 4 and 5, or one of them could be 2. If you wanted to, you could frame this at the outset.

Now to 17: If neither X nor V confessed, neither can go in slot 5. That leaves only Y to go in that slot. Y, therefore, confessed. D is the correct answer. The completed diagram would look like this:

___ -C ___ -C C C -C
Z X/V T V/X Y W S

Hope this helps!
Laura Damone
LSAT Content & Curriculum Lead | Manhattan Prep